- Joined
- Apr 2025
- Subscription
- Free
Help?
I understood the stimulus and identified the flaw correctly, but chose the incorrect AC. I eliminated A because it said at the end "both" phenomena and in my mind I felt this was correct all the way up until 'both' was stated as for me we only need to state that the observed correlation can be due to other factors which contributes to the observed phenomena .... there both seemed not accurate..... I cant seem to process the both here to make sense. I ended up choosing B, but I get now why be is wrong, I just need help with A. #feedback
If you are like me, then you got this wrong due to being confused by the word play. The below explanation from our peers didnt help me much, so after figuring it out I wanted to share my take to possibly help someone like me out there understand!
My best way to process why B is right is by rewriting the AC with simple english and replacing what is reference with what we know to make sense:
*
So, the argument "Fails to consider that [apples] have been cultivated for a short time may be similar to wid [apples] more than those from long ago." *
Essentially when I read B now for me it reads as:
It could be that outside of size there are other things in common between the cultivated and wild apples in terms of genetic makeup that isnt considered. only thing being reviewed is size. but like JT mentions we can modify the size of apples so that doesnt mean the arent the same kind of apple. So using size as the sole indicator is the problem! This identifies a flaw.
Hope this helps someone out there, as I struggled on this.
I chose the sucker choice and picked (E) but i understand while E's statement isnt wrong its pertaining to the wrong comparison. We need to focus on comparing Money to Health.
See my issue with (A) being the correct answer is that it says "Money should be acquired only if its acquisition will not make happiness unobtainable." For me, the word 'should' felt like a red flag. Who is to say we should acquire money? If it has said instead "Money can be acquired only if .... " , well then I would have considered it a viable option. Can someone please help me out here? #feedback
Please Help - I still feel uneasy about this question. I get why C, D, E are wrong. But I question A and B. See my rational below:
A- Ok maybe you could argue publishers have always been interested in making money because then it would weaken the argument, as there argument that few books are being published of intrinsic value due to money is false, since they always were about the money, so that cant explain the change.
B- (This is the correct Answer and I get it when explained but idk if I would identify this as the answer on my own) Saying a “notable decline in the quality of written books in recent years” doesn’t = " intrinsic merit" to me, and we are usually told not to assume. Also there can be low quality book which have intrinsic merit right? So how am I suppose to figure this is the correct option.
C- I get that the word “unexpectedly” is what makes this wrong. I chose this because I overlooked this word and thought that if publishers were making money on these type of books, arguing that publishers stop these type of books would weaken the rationale. But now that I see the word “unexpectedly” I get it.
D- This doesn’t weaken the Argument
E- This doesn’t weaken the Argument
So frustrated. This practice test brought my score down so much. Specifically this passage. I only got 1 correct FML. I chose the sucker AC for every Question for this passage. Like I guess I am glad that i got the alternative best AC but this really sucks. I feel like I HAVE BEEN STUDYING FOREVER.
I had D then blind review chose B. I think what tripped me up was the survey breakdown. I was confused if they were comparing two results or if it all was 1 survey because they said they tracked the changes of people's opinion so I thought it changed from 31% in favor to 38% in favor and from 24% against to unknown against with 7% refusing to answer. this left me thinking while the increase occurred we are left with major gaps for %s and we cannot conclude most. I just dont understand why they separated 31% and 38% as it doesnt make sense. Maybe someone can provide me rational ...
HELP - I'm really not understanding why A is correct. Like where is my understanding lacking?
Answer A states essentially that "Orchards depends on tress for fruit, but the trees declined and therefore fruit decline" --> To me this weakens the argument as It provides an alternative reason for why there is less fruit being ruined by insects because there is less fruit in general to be impacted by insects. Therefore it weakens the argument that the new pesticide is the working, when in reality less fruit is being grown so less fruit can be impacted.
What am I missing here, I am lost.
I need help please!!
(Question 1)
I selected (B) because I read answer choice (B) as saying that countries which have laws to enforce committees are better than countries that allow voluntary committees. (Which is true according to the stimulus Canada and Swiss are better (enforced) than US (allows voluntarily). Making this clearly supported. Yet The explanation provided interprets (B) as saying that the success level of the voluntarily committee is worse than enforced committees and we cant assume we know which was committee had better systems. This explanation makes sense why it would then be crossed off. But the way I read it was different! How should I change my approach to reading (B) and what was wrong with my interpretation?
(Question 2)
I thought (A) was wrong because there is no evidence to support that a committee would work for USA specially for medium and large companies, just because it worked in Canada and Swiss doesn't give evidence for the USA. I understand we were expected to assume that enforcing a committee would be successful for the USA. I feel its a bit contradictory to say we cross out "B" because we dont have evidence that the voluntary committee was run worse than enforced committees by Law, yet for (A) we assume that because enforced committee by Law works in Canada and Swiss for medium to large businesses it will work for the USA even though no evidence is given. Like how do you know then when to assume a connection ?
Thank you in advance for your help!
I understand why E is correct and overlooked the proportional relationship of some for all error, but I am still struggling to understand why C is wrong and how I can change my thinking so it doesn't happen again. I thought that if the criteria is ethics , the law doesn't always align to ethics, sometimes ethical decision can result in breaking the law. so I was looking for this and thought C was most closely to that. Therefore, I understood C to mean "that the argument assumes that because a certain action has a certain result (as in being something being criminal) that the person intended that result (as in the person desired to be a criminal but could be acting with accordance to their ethical standards) which would harm the argument. #feedback #Help
My reason for why I got this correct is different than J.Y. and I am not sure if my logic is sound, so I'm curious if my reason is alright as I was stuck between D and E but ultimately chose D and got it correct.
E- is a comparison: Kay agrees with more ideas than disagrees for this candidate. Yet when reading it says it is okay for Kay to select a candidate who she disagrees with on 1+ ideas WHEN she disagrees with the other candidates MORE, otherwise its unacceptable to vote. Given that E doesnt mention the other candidates we dont know if this is acceptable and cant be certain if it holds.
This question made me feel slow. I legit worked on it for like almost 30mis to an hour because while I understoof JT's rational I didn't understand how that translated to the AC D especially with 'sometimes' but Im pretty sure I figured it out and I didnt see anyone explain it in this way on the thread so im hoping if your like me this helps!
The way i am making sense of this is that routinely non-punishment is like 80% of the time not punished. The conclusion suggest never allow to go unpunished. But its not that it should "never" allow it , its that it should not allow it most times ( 80%) so if not most then some . Therefore, its okay for nonpunishment to be less than 50% (sometimes). Yet the conclusion takes it too far by saying 'never". As a result, the confusion is that the routine nonpunishment of violations of a rule (80%) should be "sometimes" and not "never". therefore confusing nonpunishment with sometimes.
I really hope this helps someone out. :)